⦁ Write an equation of a line in slope intercept form (y = mx + b) with slope 1/3 going through the point (-6, 2). Show your work for full credit.

Answers

Answer 1

[tex](\stackrel{x_1}{-6}~,~\stackrel{y_1}{2})\qquad \qquad \stackrel{slope}{m}\implies \cfrac{1}{3} \\\\\\ \begin{array}{|c|ll} \cline{1-1} \textit{point-slope form}\\ \cline{1-1} \\ y-y_1=m(x-x_1) \\\\ \cline{1-1} \end{array}\implies y-\stackrel{y_1}{2}=\stackrel{m}{\cfrac{1}{3}}(x-\stackrel{x_1}{(-6)}) \\\\\\ y-2=\cfrac{1}{3}(x+6)\implies y-2=\cfrac{1}{3}x+2\implies y=\cfrac{1}{3}x+4[/tex]


Related Questions

A scout group is having a cake sale to raise $90 for camping supplies. The troop spent $18 on ingredients for cakes. Cake slices will sell for $1 each. The equation below can be used to find the number of slices of cake the troop needs to sell to raise $90. n−18=90 How many slices of cake does the troop need to sell to raise $90?

Answers

Answer:

N=108

Step-by-step explanation:

You just have to add the $90 they need to raise plus the ingredients price to get $108.

1. A rectangular kitchen sink is 25.25 inches long, 19,75 inches wide, and
10 inches deep. Find the amount of water that can be contained in the

Answers

Answer: 4,986.875

Step-by-step explanation: Multiply 25.25 x 19.75. Then, multiply the product by 10, or shift the decimal point one place to the right.

Evaluate the following
[tex]tan15 \times \frac{cos15}{sin15} [/tex]

Answers

Answer: 1

Step-by-step explanation:

tan(15) * cos(15)/sin(15)

Use tan(x) = sin(x)/cos(x) to transform the expression:

= sin(15)/cos(15) * cos(15)/sin(15)

Cancel opposite terms on both sides:

= 1

What is the ​ IQR ​ for the data set? {31, 30, 40, 35, 48, 44, 25} Drag and drop the correct answer into the box.
A.13
B.14
C.15
D.23

Answers

Answer:

B

Step-by-step explanation:

IQR is the higher qurtile subtracted by the lower qurtile so higher is 40 and lower is 20 so 14

Hope this helps have a great day:)

what is 3.31 + 1 kilometer

Answers

Answer:

4.31 kilometer

..........................................

-9.1-3.7x=9x+9.2 rounded to the nearest tenth

Answers

Answer:

See below.

Step-by-step explanation:

-3.7x-9.1=9x+9.2

       +9.1      +9.1

-3.7x=9x+18.3

-9x    -9x

-12.7x=18.3

/-12.7   /-12.7

x=-1.44, or -1.4

-hope it helps

X=-1.4 after rounding steps in picture

What is the slope of the line that passes through the points ( − 1 , − 7 ) (−1,−7) and ( 1 , − 13 ) (1,−13)? Write your answer in simplest form.

Answers

Answer:

[tex]y = -3x -10[/tex]

Step-by-step explanation:

Slope-intercept formula: [tex]\frac{y_2-y_2}{x_2-x_1}[/tex]

Find slope of  (−1, −7) and (1, -13)

[tex]\frac{y_2-y_2}{x_2-x_1}\\\\\\frac{-13-(-7)}{1-(-1)}\\\\\\frac{-6}{2}\\\\\-3[/tex]

[tex]y = mx + b[/tex]

[tex]y = -3x +b[/tex]

[tex]y = -3x +b\\-7 = -3(-1) + b\\-7 = 3 +b\\-3 -3\\-10 = b[/tex]

[tex]y = -3x -10[/tex]

Hope this helps!

please answer thisss

Answers

Find the area of the square

Find the area of the circle

Subtract the area of the circle from the area of the square.


area of square = 10 x 10 = 100 sq. Cm

Area of circle = pi x r ^2 = 3.14 x 5^2 = 78.5 sq. Cm


area of shard region = 100 - 78.5 = 21.5 cm^2

If the shaded region is the circle (which it is),

The area of it is 78.5cm²

You can find the area of a circle by using the formula: pi • radius squared

In this case, the question states to use 3.14 for pi so our equation would look like:

3.14 • 5 • 5 (5 squared is same as 5 times5)

And that is how you get 78.5cm²

Order -4, -8.2, 6, -9, and 1 3/4 from least to greatest.

Answers

Answer:

-9, -8.2, -4, 6, 1 3/4

Step-by-step explanation:

The larger the number is if it's a negative, the lower it is (if that makes sense). For example, -9 is lower than -7.

1 3/4 = 1.75

Harry has 18 chocolates. Of the 18 chocolates, 4 of them are white chocolate. The rest are milk chocolate. Which fraction is equivalent to the fraction of chocolates that are milk chocolates?

Answers

Answer:

14/18

Since 4 out of the 18 chocolates are white, the rest will be 14 milk chocolates

and in total there are 18 chocolates

What is 65,754.59 rounded to the nearest tenth

Answers

Answer:

65,7754.6

Step-by-step explanation:

because the tenths place is the place just right of the decimal and any number that is bigger than 4 makes the number to the left round up 5 becomes 6.

Answer:

65,754.6

Step-by-step explanation:

when there is a number greater than 5 after a number you must round just add 1 to the number you have to round

for example if you have to round 1.06 to the tenths it will just be

1.1

Evaluate the expression. 0.2+0.3–0.6÷6 Write your answer as an integer or a decimal. Do not round.​

Answers

Answer:

0.4

Step-by-step explanation:

(0.2+0.3) - (0.6/6)

0.5 - 0.1

Hey there!

0.2 + 0.3 - 0.6 ÷ 6


0.2 + 0.3 = 0.50 = 0.5

= 0.5 - 0.6 ÷ 6

0.6 ÷ 6 = 0.10 = 0.1

= 0.5 - 0.1

= 0.40

= 0.4


Thus, your answer is: 0.4


Good luck on your assignment & enjoy your day!


~Amphitrite1040:)

help help

what’s the value of x

measure of c

Answers

Answer:

x = 29

Angle C = 93°

Step-by-step explanation:

Since all of the interior angles of a triangle add up to 180° and we are given the values of angle A and angle B, we can find angle C by subtracting A and B from 180.

35 + 52 = 87

180 - 87 = 93

Thus, the measure of angle C is 93°.

Now that we know the measure of angle C, we can determine the value of x.

Set up an equation:

3(x + 2) = 93

3x + 6 = 93

3x = 87

x = 29

Answer:

(a)  x = 29

(b) m∠C = 93°

Step-by-step explanation:

(a)  The sum of the interior angles of a triangle = 180°

⇒ m∠A + m∠B + m∠C = 180

⇒ 35 + 52 + 3(x + 2) = 180

⇒ 87 + 3x + 6 = 180

⇒ 93 + 3x = 180

⇒ 3x = 87

⇒ x = 29

(b) As x = 29

⇒ m∠C = 3(29 + 2)

⇒ m∠C = 3 x 31

⇒ m∠C = 93°

Volume of a sphere please help

Answers

r² x height x pi x 4 divide by 3

no word form r² x h x pi x 4/3

how would I write 5 more than 1/2 of a number as a expression?

Answers

Answer:

[tex]\frac{n}{2}[/tex] + 5

Step-by-step explanation:

-> An expression does not have an equal sign

-> Let n be "a number"

-> What I underline becomes apart of our mathematical expression in the next line;

5 more than 1/2 of a number

5 more than [tex]\frac{n}{2}[/tex]

[tex]\frac{n}{2}[/tex] + 5

-(-4)(-6)-3/5(10+15)/1/3

Answers

Answer:

[tex]-29[/tex]

solving steps:

[tex]\sf \hookrightarrow -\left(-4\right)\left(-6\right)-3/5\left(10+15\right)/1/3[/tex]

[tex]\sf \hookrightarrow -\left(-4\right)\left(-6\right)-3/5\cdot \:25/1/3[/tex]

[tex]\sf \hookrightarrow -24-3/5\cdot \:25/1/3[/tex]

[tex]\sf \hookrightarrow -24-5[/tex]

[tex]\sf \hookrightarrow -29[/tex]

Answer:

the answer is 29.

Step-by-step explanation:

use the order of operations to solve
PEMDAS

Using the following​ diagram, solve for x. Show work

Answers

Answer:

x = 7.5

Step-by-step explanation:

A line parallel to a side of a triangle and intersecting the other 2 sides, divides those sides proportionally, that is

[tex]\frac{8}{10}[/tex] = [tex]\frac{6}{x}[/tex] ( cross- multiply )

8x = 60 ( divide both sides by 8 )

x = [tex]\frac{60}{8}[/tex] = 7.5

Two similar triangles have a corresponding sides of length 7cm and 4cm respectively. Find the ratio of their areas.

Answers

Answer:

49 : 16

Step-by-step explanation:

ratio of their sides = 7 : 4

ratio of their areas = 7^2 : 4^2 = 49 : 16

the population of town increases by5℅ every year. In the last year, the population was 80,000. 1) find the present population 2) find the population of the town in the next two years.


explain step by step please​

Answers

Answer:

last year it was 80k

Step-by-step explanation:

Population increases by 5% every year

Last year it was 80,000

PRESENT POPULATION

Present population = 80,000 + 5% off 80,000

                               = 80,000 + 5/100 × 80,000

                               = 80,000 + 5 × 800

                               = 80,000 + 4,000

                               = 84,000

AFTER ONE YEAR

After this , the population again increases by 5%

Population = 84,000 + 5% off 84,000

                 = 84,000 + 5/100 × 84,000

                 = 84,000 + 5 × 840

                 = 84,000 + 4,200

                 = 88,200

ANSWERS

The population of present year is 84,000

After one year , the population becomes 88,200

Hope it helps :-)

Answer:

1. The Present population is 84,000

2. The next years population 88,200 and the year after that would be 92,610

Step-by-step explanation:

If you take the previous year and multiply it by 1.05 you you get the current population. After that you take the current population and multiply it again by 1.05 since the population increases by 5% so continue multiplying by 1.05 to get the later years and so forth

What property does Line A use?
A. distributive property
B. commutative property
C. addition property of equality
D. division property of equality
E. subtraction property of equality
F. multiplication property of equality
Also what step could be used to solve the equations? The steps are 3x/3 -20= 10/3 3 · 3x-20=10 · 3 3x-20+20=10-20 or 3x-20+20=10+20 ​

Answers

Answer:

B. Commutative property

Step-by-step explanation:

In commutative property, changing the order of the operands will not change the result.

5 + 2 = 7  &

2 + 5 = 7

6. Given that ABCD is a quadrilateral with A(-2,-2), B(1, 1),
C(1,6), and D(-2, 3), determine if it is a parallelogram.

Answers

Answer:

it is a parallelogram

Step-by-step explanation:

so first plot out points and then you find the slope for each of the corresponding lines.

First lets find the slope for AB. you must subtract them and make it -2-1/-2-1. it ends up having a slope of 1.

Then find the slope of the corresponding side(CD). this we will subtract -2-1/3-6. this will have a slope of 1 and will be parallel to AB.

Next you find the slopes of the other two sides starting with AC. we will subtract -2-(-2)/-2-3. this will be a vertical line which means it has a slope of 0.

finally the last step is you see if BD has an equal slope to AC. you then subtract 1-1/1-6. this is also a vertical line so the slope is 0 just like AC so they are both parallel.

in conclusion there are two sets of parallel lines so you have a parallelogram

9. V = 300(1.02)* models exponential
growth, where t represents the number
of years.
a) What is the annual rate of growth?

b) Calculate the monthly rate of growth
to the nearest hundredth of a percent.

Answers

Answer:

  a) 2%

  b) 0.17%

Step-by-step explanation:

Exponential growth is modeled by the equation ...

  y = a·b^x

where 'a' is the initial value, and 'b' is the growth factor.

When x is has units of time, the growth factor 'b' applies over 1 unit of x. The applicable growth factor for different units of time can be found using the rules of exponents applied to the equation written with x expressed in the different units.

The growth rate is related to the growth factor by ...

  b = 1 +r . . . . where r is the growth rate

  r = b -1 . . . . solved for r

__

a)

Your equation is ...

  V = 300(1.02^t)

where t is expressed in years. Then the corresponding annual growth rate is ...

  b = 1.02

  r = b -1

  r = 1.02 -1 = 0.02 = 2% . . . annual rate of growth

__

b)

When the equation is written so that t is expressed in months, it becomes ...

  V = 300(1.02)^(t/12)

  V = 300(1.02^(1/12))^t

Now, the growth factor (per month) is

  b = 1.02^(1/12)

and the monthly growth rate is ...

  r = 1.02^(1/12) -1 ≈ 1.00165158 -1 = 0.00165158

  r ≈ 0.17% . . . . monthly rate of growth

A roll of wire is cut into 12 pieces. EACH piece is 3/4 meter long. What is the total length of the wire on the roll????? ​

Answers

Answer:

9m

Step-by-step explanation:

3/4 = 0.75

each piece = 0.75

0.75x12=9

Answer:

9 meters.

Step-by-step explanation:

Because there are 12 pieces that were cut.

If each were 3/4 meters then what is 3/4 x 12= 9 meters and that were the length

Have a great afternoon Hope this helps:)

PLEASE HELP ASAP!!!!!!

Answers

Answer:

[tex]\frac{1}{axy}[/tex]

Step-by-step explanation:

[tex]\frac{xy}{a^2+a^3}[/tex] × [tex]\frac{a+a^2}{x^2y^2}[/tex] ( factorise denominator and numerator of 2 fractions )

= [tex]\frac{xy}{a^2(1+a)}[/tex] × [tex]\frac{a(1+a)}{x^2y^2}[/tex]

Cancel (1 + a) from both fractions

= [tex]\frac{xy}{a^2}[/tex] × [tex]\frac{a}{x^2y^2}[/tex]

Cancel a and xy from both fractions

= [tex]\frac{1}{a}[/tex] × [tex]\frac{1}{xy}[/tex]

= [tex]\frac{1}{axy}[/tex]

a ≠ 0 , x ≠ 0 , y ≠ 0

as this would would make the function undefined

Answer:

Step-by-step explanation:

[tex]\dfrac{xy}{a^2+a^3}\times \dfrac{a+a^2}{x^2y^2}[/tex]

Apply the fraction rule [tex]\dfrac{a}{b}\times \dfrac{c}{d}=\dfrac{a \times c}{b \times d}[/tex] :

[tex]\implies \dfrac{xy(a+a^2)}{x^2y^2(a^2+a^3)}[/tex]

Cancel the common factor [tex]xy[/tex] :

[tex]\implies \dfrac{(a+a^2)}{xy(a^2+a^3)}[/tex]

Factor [tex](a+a^2)=a(1+a) \ \ \textsf{and} \ \ a^2+a^3=a^2(1+a)[/tex] :

[tex]\implies \dfrac{a(1+a)}{xy \cdot a^2(1+a)}[/tex]

Cancel the common factor [tex]a(1+a)[/tex] :

[tex]\implies \dfrac{1}{axy}[/tex]

I think the "if" part is a, x and y cannot equal zero, as otherwise the expression will be undefined.

So I would put: [tex]x\neq 0; y\neq 0, a\neq 0[/tex]

Ari has two pieces of ropes, a and b, their lengths are in the ratio 3:5. rope a is 90 cm long. if ari cuts of the ratio of the length of rope a to the remaining length to rope b?​

Answers

Answer:

This is ratio. A:B = 3:5

If the 3 parts of a =90 that means 1 part =30 (90/3=30).

So B =5.

5 parts would equal 30x5= 150

B=150cm

Answer:

Ari has two pieces of ropes, a and b, their lengths are in the ratio 3:5. rope a is 90 cm long. if ari cuts of the ratio of the length of rope a to the remaining length to rope b?​

Step-by-step explanation:

7.4
Practice A
In Exercises 1-5, the diagonals of rhombus ABCD intersect at E. Given that
mZEAD - 67', CE - 5, and DE 12, find the indicated measure.
1. MZAED
5
2. MZADE
E
12
3. MZBAE
67
44E
5. BE
In Exercises 6 and 7, find the lengths of the diagonals of rectangle JKLM.
6. JL = 3r + 4
7. JL = 2x - 6
KM = 4x - 1
KM
*+1
In Exercises 8 and 9, decide whether quadrilateral WXYZ is a rectangle, a
rhombus, or a square. Give all names that apply. Explain your reasoning.
8. W(3, 1), X(3,-2), Y(-5, -2), 2(-5, 1) 9. W(4, 1), X (1,4), Y(-2, 1), Z(1, -2)
Р
o
R
a
10. Use the figure to write a two-column proof.
Given: PSUR is a rectangle.
PQ E TU
Prove: OS = RT
S
U
11. In the figure, all sides are congruent and all angles are right angles.
B
a. Determine whether the quadrilateral is a rectangle.
Explain your reasoning.
b. Determine whether the quadrilateral is a rhombus.
Explain your reasoning
c. Determine whether the quadrilateral is a square.
Explain your reasoning.
d. Find m2AEB.
D
e. Find mZEAD.

Answers

The properties of the quadrilaterals gives the equivalent relations used

to find the lengths and the angles.

Response:

1. m∠AED = 90°

2. m∠ADE = 23°

3. m∠BAE = 67°

4. AE = 5

5. BE = 12

6. 19

7. 22

8. WXYZ is a rectangle

9. WXYZ is a square

10. [tex]\overline{QS} \cong \overline{RT}[/tex] by Corresponding Parts of Congruent Triangles Congruent, CPCTC

11. a. The diagonals are perpendicular and the given figure is not a rectangle

b. The given figure is not a rhombus

c. The figure is a square.

d. ∠AEB = 90°

e. m∠EAD = 45°

Which properties of a quadrilateral can be used to find the  required dimensions?

1. The  diagonals of a rhombus bisect each other at right angles

Therefore;

m∠AED = 90° (by definition of right angles)

2. m∠EAD and m∠ADE are complementary angles

Which gives;

m∠EAD + m∠ADE = 90°

m∠ADE = 90° - m∠EAD

Therefore;

m∠ADE = 90° - 67° = 23°

3. The diagonals of a rhombus bisect the angles, therefore;

m∠BAE = m∠EAD = 67°

4. The diagonals bisect each other, therefore;

AE = CE = 5

5. BE = DE = 12

6. JL = 3·x + 4

KM = 4·x - 1

Which gives;

3·x + 4 = 4·x - 1

4·x - 3·x = 4 + 1 = 5

x = 5

KM = JL = 3 × 5 + 4 = 19

The lengths of the diagonals of rectangle JKLM is 19

7. JL = 2·x - 6

[tex]KM = \mathbf{\dfrac{3}{2} \cdot x+ 1}[/tex]

Which gives;

[tex]\frac{3}{2} \cdot x+ 1 = 2\cdot x - 6[/tex]

[tex]2\cdot x -\frac{3}{2} \cdot x = 6 + 1 = 7[/tex]

[tex]\dfrac{1}{2} \cdot x = 7[/tex]

x = 2 × 7 = 14

JL = 2 × 14 - 6 = 22

The lengths of the diagonals of rectangle JKLM  are 22

8. W(3, 1), X(3, -2), Y(-5, -2), Z(-5, 1)

WX = 1 - (-2) = 3

YZ = 1 - (-2) = 3

XY = 3 - (-5) = 8

WZ = 3 - 5 = 8

Slope of WX = (1 - (-2)) ÷ (3 - 3) = ∞

Slope of YZ = (1 - (-2)) ÷ (-5 - (-5)) = ∞

Slope of XY = (-2- (-2)) ÷ (3- (-5)) = 0

Slope of WZ = (1- 1) ÷ (3- (-5)) = 0

Therefore;

WX and YZ are perpendicular to XY and WZ

The properties of WXYZ are the properties of a quadrilateral having perpendicular sides.

Given that the sides are not equal, the figure is a rectangle

9. W(4, 1), X(1, 4), Y(-2, 1), Z(1, -2)

WX = √((4 - 1)² + (1 - 4)²) = 3·√2

YZ = √((1 - (-2))² + (-2 - 1)²) = 3·√2

XY = √((1 - (-2))² + (4 - 1)²) = 3·√2

WZ = √((1 - (-2))² + (4 - 1)²) = 3·√2

Slope of WX = (4 - 1) ÷ (1 - 4) = -1

Slope of YZ = (-2 - 1) ÷ (1 - (-2)) = -1

Slope of XY = (1 - 4) ÷ ((-2) - 1) = 1

Slope of WZ = (-2 - 1) ÷ (1 - 4) = 1

Therefore;

The lengths of the sides are equal and the sides are perpendicular to each other

Therefore;

WXYZ is a square

10. The two column proof is presented as follows;

Statement [tex]{}[/tex]                                            Reason

PSUR is a rectangle [tex]{}[/tex]                           Given

[tex]\overline{PR} = \overline{SU}[/tex] [tex]{}[/tex]                                             Opposite sides of a rectangle

[tex]\overline{PQ} \cong \overline{TU}[/tex]  [tex]{}[/tex]                                           Given

[tex]\overline{PQ} = \overline{TU}[/tex]  [tex]{}[/tex]                                           Definition of congruency

[tex]\overline{PR} = \overline{RQ} + \overline{PQ}[/tex] [tex]{}[/tex]                                  Segment addition postulate

[tex]\overline{SU} = \overline{ST} + \overline{TU}[/tex] [tex]{}[/tex]                                   Segment addition postulate

[tex]\overline{RQ} = \overline{ST}[/tex]    [tex]{}[/tex]                                          Addition property of equality

ΔPQS ≅ ΔTUR    [tex]{}[/tex]                                ASA rule of congruency

[tex]\overline{QS} \cong \overline{RT}[/tex]         [tex]{}[/tex]                                    CPCTC

11. The properties of a rectangle are;

The interior angles are 90°

The diagonals are not perpendicular to each other

In the given figure, the four triangles formed by the diagonals are congruent, therefore;

The angles at the vertex point of the four tringles are equal to each other and therefore, equal to 90°

Therefore;

The diagonals are perpendicular and the given figure is not a rectangle

b. The lengths of the diagonals, are equal, therefore;

The given figure is not a rhombus

e. The properties of the figure, which includes;

All sides are equal

The interior angles are 90°

The diagonals are equal and bisect each other at 90° indicate that the figure is a square

d. The diagonals of a square are angle bisectors of the interior angles

Therefore;

m∠ABE = m∠CBE

m∠ABE + m∠CBE = 90° by definition of complementary angles

Therefore;

m∠ABE  = 90° ÷ 2 = 45°

Similarly;

m∠ABE = m∠BAE = 45°

m∠AEB = 180° - (m∠ABE + m∠BAE)

m∠AEB = 180° - (45° + 45°) = 90°

e. m∠EAD = m∠BAE = 45° (angles formed by the diagonal AC)

Learn more about quadrilaterals here:

https://brainly.com/question/12635945

ARE THESE Equivalent?    6:7   &   24:28  HELP WILL GIVE BRAINLIST IF CORRECT AND GOOD ANSWER

Answers

YesBecause they are divisible by both numbers in the first ratio.

Technically any two numbers in a ratio that are divisible by the other ratio numbers make them equivalent.

For instance,

[tex]3:6[/tex]

[tex]6:12[/tex]

[tex]Equivalent.[/tex]

Because...

[tex]\frac{3 * 2 = 6}{6 * 2 = 12}[/tex]

Or...

[tex]\frac{3}{6} = \frac{6}{12}[/tex]

The equation above represents, "Divide 3 by 6 and divide 6 by 12". From both equations to here, you now know what to multiply the numbers by to get your answer.

From here, multiply 3 by 2, and, 6 by 2, to get the equivalent ratio, 6:12.

Therefore, 6:7 and 24:28 are equal as 6:7 and 24:28 are, too.

Which shows the algebraic representation of the phrase and its value when n = 4. 2? ""twenty-eight plus the product of five and a number"" 28 minus 5 n; when n = 4. 2 the value is 7 28 5 n; when n = 4. 2 the value is 49 28 n minus 5; when n = 4. 2 the value is 112. 6 28 n 5; when n = 4. 2 the value is 122. 6.

Answers

The algebraic representation of the phrase is 28+5n and when n  is 4.2, then the value is 49.

What is algebraic expression?

Algebraic expression are the expression which consist the variables, coefficients of variables and constants.

The algebraic expression are used represent the general problem in the mathematical way to solve them.

Steps to write down the simple algebraic expression-

Identify all the factors of the expression.Assume a variable for the changing values.Put a coefficient for the variable, if there is any rate of change of variable.Use constant numbers for the fixed values.

The statement given in the problem is that the twenty-eight plus the product of five and a number.

Let suppose the unknown number is n. Now the product of this number with 5 can be written as,

[tex]5\times n\\5n[/tex]

As, the equation is made when the number 28 is plus in this product. Thus the algebraic representation of the phrase is,

[tex]28+5n[/tex]

When the value of n is 4.2, then the value of this algebraic would be,

[tex]28+5(4.2)\\28+21\\49[/tex]

Hence, the algebraic representation of the phrase is 28+5n and when n  is 4.2, then the value is 49.

Learn more about the algebraic expression here;

https://brainly.com/question/2164351

If C = t - 5 and B = t^2 + 2, find an expression that equals 2C + 2B in standard
form.

Answers

Answer:

t² + t - 3 = 0

Step-by-step explanation:

C = t - 5

B = t² - 2

2C + 2B

= 2(t - 5) + 2(t² + 2)

= 2t - 10 + 2t² + 4

= 2t² + 2t - 6

=> t² + t - 3 = 0

Find the distance between each pair of parallel lines with the guven equation y=2x+7 y=2x-3

Answers

Answer:

10

Step-by-step explanation:

The y-intercept for each line is...

y=2x+7 (0, 7)

y=2x-3 (0, -3)

Note this is because...

y=mx+b

b represents the y-intercept

Using the distance formula...

[tex]d=\sqrt{(x_2-x_1)^2+(y_2-y_1)^2[/tex]

[tex]d=\sqrt{(0-0)^2+(7-(-3))^2[/tex]

[tex]d=\sqrt{0+(10)^2[/tex]

[tex]d=\sqrt{0+100}[/tex]

[tex]d=\sqrt{100}[/tex]

[tex]d=10[/tex]

Other Questions
How does the author develop the narrators point of view in paragraphs 20-22 in the passage Popularity by Adam Bagdasarian Why did Nazi Germany build concentration camps in Poland? Find an equation of the line. Write the equation in standard form.Through (10,25) and (4, 13) -9-12 equivalent equation True or false. A direct current moves through a circuit in only one direction.TrueFalseI'm 99% sure it's true. write a ratio in three different forms. Shawn biked 5 miles and ran 2 miles. What is the ratio of the distance Shawn ran to the distance he biked? OTHER THAN 2:5 What is an overall problem with all of the different city models. Which of the following expressions is equivalent to 14xy 28x - 36y + 48? what's the multiplication table The international role of the United States changed as a result of World War I what was the short term effect of this change a return to isolationism AC= 2x-7, BC= x-3, AB= 8. find x Please solve for X and find the measure of B. Please answer it for me, its in Spanish! Which of the following statements is false?(A) At most temperatures, both plants consumedcarbon dioxide.(B) Both plants responded to changes in lightintensity in the same way.(C) Both plants responded to changes in tempera-ture in the same way.(D) Carbon dioxide has been used to producesugars and many other molecules needed bythe plants. fossils can tell us about the organism and what else ?(a) How the earth was before?(b) The weather(c) The number of babies The majority of retailers are:A. Independently ownedB. CorporationsC.FranchisesD Online Only Europeans were interested in west africa especially for its. Answer quick please answer 5 and 3! Select the correct answer. What is the probability that the manufacturing unit has carbon emission beyond the permissible emission level and the test predicts this? A. 0. 2975 B. 0. 0525 C. 0. 0975 D. 0. 5525 E. 0. 6325. Olympia ate lunch at a restaurant. The amount of her check was $6. 89. She left $8. 00 on the table, which included the amount she owed plus a tip for the waiter. Which equation shows t, the amount of her tip, in dollars? 6. 89 t = 8. 00 6. 89 - t = 8. 00 6. 89t = 8. 00 6. 89 = 8. 00 Divided by t.